Course menu
Course Menu
Lesson complete ✅
Click here to view the next lesson.

3.5 - Must Be True

Raise Hand   ✋

Must Be True questions (MBTs) ask us to make inferences based on the passage. In other words, to find what else must be the case based on the available information.

MBTs tend to look like this:

  • Which one of the following can be properly inferred?
  • If the columnist's statements are true, then which one of the following must also be true?
  • If the statements are true, which one of the following cannot be true?

Sometimes, you'll get a list of facts that point to something else when strung together. Be careful. It's easy to fall into the outside knowledge trap on these questions. Stick to the passage alone.

Other times, these play out like mini Logic Games, hitting you with wordy conditional logic you need to tease apart.

In either case, the method's the same. Consider each fact or condition on its own merits, tying it into the others as you go.

Examples

PrepTest 73, Section 2, Question 21

Check out PrepTest 73, Section 2, Question 21:

Most of the students who took Spanish 101 at the university last semester attended every class session. However, each student who received a grade lower than B minus missed at least one class session.

Let's simplify things.

The first sentence tells us that more than half of Spanish 101-takers attended every class session. That's more than 50% (and, at this point, possibly all) Spanish 101 students going to every single class. Got it.

The next sentence tells us that each (every) student who finished below B- missed at least one class session. This removes the earlier possibility that majority could still mean all. It also tells us that, since more than 50% of Spanish 101 students had perfect attendance, they must have gotten a B- or higher. Had they not, they'd be part of the group of students scoring below B-, each of whom missed one or more classes.

That's it. Those are the things that must be true. We're heading into the answer choices armed to the teeth.

June 2007 PrepTest, Section 3, Question 22

Next, we're looking at June 2007 PrepTest, Section 3, Question 22:

If the price it pays for coffee beans continues to increase, the Coffee Shoppe will have to increase its prices. In that case, either the Coffee Shoppe will begin selling noncoffee products or its coffee sales will decrease. But selling noncoffee products will decrease the Coffee Shoppe's overall profitability. Moreover, the Coffee Shoppe can avoid a decrease in overall profitability only if its coffee sales do not decrease.

This one's a mouthful of conditional logic. Take it one step at a time.

First condition: If bean prices go up, then shop prices go up. Simple enough.

Second condition: If shop prices go up, then either it will start selling noncoffee products or its coffee sales will decrease. We can string this together with the first sentence. If bean prices go up, then the shop will either sell noncoffee products or lose some coffee sales.

Third condition: If the shop sells noncoffee products, then its overall profitability will decrease. This ties all the way back to the very first condition and accounts for one of the two possible paths presented in the second condition. In other words, if bean prices go up, and the shop goes the noncoffee product route, its overall profitability will go down.

Fourth condition: If the shop avoids a decrease in overall profitability, then its coffee sales do not decrease. This means if we lose coffee sales, profits will drop. Like the third condition, this fourth one ties all the way back to the first. If bean prices go up, therefore shop prices go up, and the shop elects not to sell noncoffee products, then coffee sales go down, resulting in lost profits.

Sounds like our coffee shop is pretty screwed if bean prices go up—all paths lead to lower overall profitability. That's what must be true.

Really, any of the teased apart conditions could constitute something that must be true. We're completely prepared, no matter what the answer choices throw our way.

---

That's it for the LR version of Must Be True. Next, we're tackling Most Strongly Supported questions.

0 Comments

Active Here: 0
Be the first to leave a comment.
Loading
Someone is typing...
No Name
Set
4 years ago
Admin
(Edited)
This is the actual comment. It can be long or short. And must contain only text information.
No Name
Set
2 years ago
Admin
(Edited)
This is the actual comment. It's can be long or short. And must contain only text information.
Load More
Thank you! Your submission has been received!
Oops! Something went wrong while submitting the form.
Load More
Leave a comment
Join the conversation
You need the Classroom Plan to comment.
Upgrade